A Brilliant Limit

A Brilliant Limit

blackpenredpen

6 лет назад

1,410,707 Просмотров

Ссылки и html тэги не поддерживаются


Комментарии:

Shaurya Chaturvedi
Shaurya Chaturvedi - 14.11.2023 15:56

Such an irony I just solved this question couple hours ago😂

Ответить
himanshu
himanshu - 13.11.2023 18:05

Can we use the Stirling approximation of factorials for this?

Ответить
TheFlightCombatant
TheFlightCombatant - 13.11.2023 17:23

indeed it is brilliant

Ответить
F. Nils
F. Nils - 13.11.2023 00:12

got a math test tomorrow, hope it will help ( yes I rather look at a video i know nothing about ouvre working )

Ответить
Ayoub Boumhaout
Ayoub Boumhaout - 08.11.2023 22:38

The part with riemann sum is just genius

Ответить
Jakub Frei
Jakub Frei - 08.11.2023 07:51

This has to be one of the most beautiful math ive ever seen

Ответить
Asit Patel
Asit Patel - 07.11.2023 11:26

This topic is taught to 12th grade students in india

Ответить
Prof ENK
Prof ENK - 05.11.2023 15:29

Absolutely brilliant

Ответить
Prof ENK
Prof ENK - 05.11.2023 15:28

Brilliant…

Ответить
Neon Lines
Neon Lines - 04.11.2023 23:16

What an absolutely brilliant solution! This is one of the coolest limits I've ever seen, thank you so much for showing this to the world!

Ответить
Bogdan Garkusha
Bogdan Garkusha - 04.11.2023 17:54

thank you for amazing explanation 😊

maybe a stretch, it looks like it can be solved by taking that root

lim{n->°°}(n!/n^n)^(1/n) = lim{n->°°}(n!^(1/n)/n) = lim{n->°°}1*(2^(1/n))*...*((n-c)^(1/n))*...*(n-1)^(1/n)) all of the constants turn into 1, therefore lim{n->°°}((n-c)^(1/n))*...*(n-1)^(1/n)) ~ lim{n->°°}((n-c)^(c/n)) = lim{n->°°}((n+k)^(-k/n)) = lim{n->°°}(n(1+(k/n))^(n/k)) = e^k*lim{n->°°}(n)^(n/k)) = e^k = (e^-c) = 1/e^c

Ответить
Joel MacInnes
Joel MacInnes - 28.10.2023 00:49

Why does e keep showjng up everywhere

Ответить
FrogSuffer
FrogSuffer - 25.10.2023 18:52

using my methods i just found out that ((loggamma(n+1) - n * log(n)) * (1 / n)) the bigger n is the closer it is to -1 and exp(-1) is e ^ -1 which is 1 / e
so the ans. is 1 / e

Ответить
Slash
Slash - 14.10.2023 20:10

That was the question my high school teacher gave me to solve as the first question of definite integral as a limit of sum

Ответить
Akash Singh
Akash Singh - 05.10.2023 19:27

The only thing the title was missing was the exclamatory sign ,btw sir your videos really changed my thought process thank u ❤

Ответить
Nick Paras
Nick Paras - 05.10.2023 01:12

And why did you use ln and not a general log?

Ответить
Daniloy
Daniloy - 08.09.2023 03:13

I may be mistaken, but wouldn’t ln(n!/n^n) be a discrete function if we use the normal definition of factorials and not the gamma function definition? Because then you can’t just plug any value for n in 1/n.(ln(1/n) + ln(2/n) + … ln(n/n))

Ответить
Phileas Mahuzier
Phileas Mahuzier - 05.09.2023 23:18

just use stirling lol...but riemann series works as well

Ответить
Cappello M
Cappello M - 25.08.2023 11:06

Hai fatto capire benissimo la tua esperienza esilarante. Complimenti per il video!

Ответить
Monika
Monika - 07.08.2023 12:09

we can use Stirling's Formula approximation for n!. as the limit tends to infinity n! ~ √(2πn)(n/e)^n

Ответить
Marius Principaud
Marius Principaud - 04.08.2023 01:45

Become very easy with stirling formula lmao 😅

Ответить
Coby Ambrose
Coby Ambrose - 29.07.2023 00:44

Does this work with any logarithm base or does it have to be the natural log?

Ответить
Learner 🦄
Learner 🦄 - 28.07.2023 09:52

🥺wooww💝

Ответить
Dhruv...
Dhruv... - 18.07.2023 14:55

You could easily use L'Hopital... (ln(1/n) + ln(2/n)....)/n is the infinity upon infinity form

Ответить
Franciescio Rino
Franciescio Rino - 02.07.2023 19:35

This is a very beautiful limit 😍

Ответить
Dominexis
Dominexis - 16.06.2023 04:57

I had a big smile when you made the connection between the series and a Riemann sum.

Ответить
Hrishikesh Vinod
Hrishikesh Vinod - 08.06.2023 06:26

This beautiful limit is literally like one of the starting questions from the infamous black book of Kota

Ответить
PingalPie
PingalPie - 04.06.2023 15:07

Multiply power to both numerator and denominator which makes denominator 'n', so limit will become zero

Ответить
Daniel Parr
Daniel Parr - 25.05.2023 21:23

The no nut november equation

Ответить
Edward Hudson
Edward Hudson - 24.05.2023 20:34

does this even work though since factorial requires n to be an integer, so n cant continuously increase as required for an integral

Ответить
С-8КОМ
С-8КОМ - 23.05.2023 17:22

You can solve the integral of ln x from 0 to 1 easily by using the inverse function of ln x. The area is actually equal to the negative of the area under the curve of e^x from -infinity to 0, because they are inverse functions, so you can solve that easily to get -1.

Ответить
Athey Bengala
Athey Bengala - 23.05.2023 05:46

What? But n∈ℕ, what are you taking the area from???

Ответить
Miha Leben
Miha Leben - 21.05.2023 18:58

Why
Why would you do this

Ответить
Wrook
Wrook - 21.05.2023 05:13

For the first step, if you took the logarithm of both sides with a base other than e, would the answer still work out to be 1/e? (For example, if you took log base 5 of both sides initially instead of ln)

Ответить
Παντελεημων Προδρομιδης
Παντελεημων Προδρομιδης - 19.05.2023 12:20

Να εφαρμοσουμε Stolz υπολογιζεται πιο γρηγορα και ευκολα

Ответить
Simba
Simba - 19.05.2023 02:17

Stirling's formula enables us to calculate the limit quickly :)

Ответить
Thundercraft049
Thundercraft049 - 13.05.2023 08:55

I got also 1/e

I just know that when the function n/(n!)^(1/n) goes to infinity is e

And the function (n!/n^n)^(1/n) can be simplified into ((n!)^(1/n))/n

Now the function is similar to the other limit but just being he reciprocal

Ответить
Bell
Bell - 12.05.2023 00:16

If L was being multiplied by ln, why didn’t he divide the right side by ln instead?

Ответить
Utku Gündoğdu
Utku Gündoğdu - 01.05.2023 16:03

götlükler ezberimde

Ответить
mrpsychodeliasmith
mrpsychodeliasmith - 18.04.2023 00:01

Why not use the Gamma fumction instead of n! - you never said!

Ответить
Marckie Zeender
Marckie Zeender - 06.04.2023 08:45

Didn't watch the video, but based on Stirling's approximation I'm gonna say it's equal to e

Ответить
Zlad Tv
Zlad Tv - 02.04.2023 14:20

I used the Stirling formula; n!~sqrt(2*pi*n)*((n/e)^n), and i got e^-1 as well.

Ответить
Techno Coderz🔥🔥
Techno Coderz🔥🔥 - 26.03.2023 09:16

I thought the result would be 1 because 1/infinity leads to 0 and any number raised to zero is 1

Ответить
Yan QUEVAREC
Yan QUEVAREC - 13.03.2023 15:37

I didn't understand why the area under the curve is equal to minus one.

Ответить
장종윤
장종윤 - 23.02.2023 11:27

Good

Ответить